Intégrabilité de 1 par rapport à la mesure Px

Bonjour la communauté,

Une question par rapport à la preuve ci-jointe de ce théorème:
La fonction caractéristique est continue sur $\mathbb{R}^N $
avec la fonction caractéristique définie par $\varphi_X(t)=\mathbb{E}(e^{i<t,x>})$.

Je ne comprends pas le passage $1$ est intégrable par rapport à la mesure $P_X$, cette dernière étant la loi de $X$.
Il me semble que la notion d'intégrabilité d'une fonction $f$ au sens de Lebesgue dépend des conditions:
- $f$ est mesurable
- L'intégrale impropre converge, ce qui veut dire qu'on ne se restreint pas à un compact de l'ensemble de définition de $f$
Or pour $f=1$, elle n'est pas intégrable sur $\mathbb{R}$

Merci pour vos lumières.74118

Réponses

  • Pas intégrable par rapport à la mesure de Lebesgue, mais intégrable par rapport à n'importe quelle mesure de probabilité.
  • La fonction $f$ constante égale à $1$ n'est pas intégrable pour la mesure de Lebesgue mais elle l'est pour toute mesure finie, en particulier toute mesure de probabilité : $\int_\R f(x)P_X(\mathrm{d}x)=P_X(\R)=1$.
  • Merci beaucoup Alea et Math Coss

    Pourriez-vous me donner un lien ou un théorème pour que je puisse comprendre pourquoi la fonctions constante n'est pas Lebesgue-intégrable ?

    Serait-ce parce que $ \int_{-\infty}^{+-\infty} 1 d \lambda = \lambda (\mathbb{R}) = +\infty - (-\infty) = +\infty $ ?
    Avec $ \lambda $ la mesure de Lebesgue.
  • Oui, tout simplement. Revois la définition de l'intégrabilité d'une fonction positive vis-à-vis d'une mesure. Au passage la notation $+\infty - (-\infty)$ est plus qu'à éviter.
  • De façon générale, l'intégrale de la fonction indicatrice $\mathbf{1}_A$ d'une partie $A$ dans un espace mesuré $(X,\mathscr{T},\mu)$, c'est par définition $\int_X\mathbf{1}_A\mathrm{d}\mu=\mu(A)$. Il devrait être assez clair que la fonction constante égale à $1$ définie sur un ensemble $X$ est l'indicatrice de $X$. Voilà pourquoi $\int_X1\mathrm{d}\mu=\mu(X)$.

    Pourquoi est-ce que la mesure de Lebesgue $\lambda$ de $\R$ vaut l'infini ? Par exemple parce que $\R$ contient $[0,n]$ pour tout entier naturel $n$ et donc $\lambda(\R)\ge\lambda\bigl([0,n]\bigr)=n$ pour tout $n$.
  • Merci beaucoup Poirot et Math Coss pour ces amples explications. C'est tout clair maintenant.

    Bonne journée à vous!
Connectez-vous ou Inscrivez-vous pour répondre.